LSAT and Law School Admissions Forum

Get expert LSAT preparation and law school admissions advice from PowerScore Test Preparation.

 ChicaRosa
  • Posts: 111
  • Joined: Aug 23, 2016
|
#31518
I don't understand why E is correct and not C?

I had a hard time trying to find the conclusion of the stimulus. I'll try to rephrase the stimulus in my own words as much as possible.

Premise: It has been argued that the public should boycott the adverstisers of shows that promote violence and destroy our country's values even though the government shouldn't censor TV shows.

Conclusion: By boycotting it is still considered censorship.

Premises: If the public boycotted the advertisers then they would cancel their ads which will cause some shows to go off the air and would lead to the restriction of shows that the public can view.

Do I have this correct?

I thought it was B because as a result of boycotting it would force some shows off the air and would restrict what the public can see which I thought implied shows that don't promote the traits stated in the stimulus.

Thanks!
 Adam Tyson
PowerScore Staff
  • PowerScore Staff
  • Posts: 5191
  • Joined: Apr 14, 2011
|
#31526
The goal here, Chica, is to justify the conclusion, which you correctly identified as "boycotting is censorship". Focus on proving that claim, that a boycott is, in fact, censorship.

Answer E does that by helping to define the scope of what censorship is. If anything that forces a show off the air is censorship, and boycotting does that (according to the premises), then boycotting must be censorship.

You asked about answer C, but then talked about liking answer B, so I am assuming the reference to C is a typo and that you were interested in B. Answer B tells us nothing about whether a boycott is the same as, or is a type of, censorship. It tells us that boycotts can force all sorts of things off the air, regardless of content. Is that censorship? I have no idea, because the stimulus didn't define censorship for us. Only answer E does that. Maybe forcing a show off the air, other than by government action, is not censorship, but is just normal market forces at work, part of a free and fair capitalistic society? Maybe there is another name for that, other than censorship?

Remain focused on the conclusion that you want to justify. If you want to prove that a boycott is a type of censorship, pick an answer that makes that connection for you, clearly and unambiguously.

I hope that helped! Good luck with your continued studies!
 LSATUSERNAME3
  • Posts: 3
  • Joined: Mar 08, 2020
|
#96330
Why is (D) incorrect. I felt that if negated, choice (D) would weaken the argument. I think the negation would be like a Chapelle situation in which some people do boycott it, but it's not restricted because of a lack of widespread agreement. In the stimuli, it says the public and I think the word public means a general consensus. Hence, I thought for a valid conclusion a general consensus among which show promotes violence or erodes values should be needed. In addition, I thought (D) was a defender assumption even though it is a justify the conclusion question.

Also, I understand (E) is correct because there's a gap between consensus and restriction in the argument. However, I was stuck between the two.
User avatar
 katehos
PowerScore Staff
  • PowerScore Staff
  • Posts: 184
  • Joined: Mar 31, 2022
|
#96387
Hi LSATUSERNAME3!

You mentioned negating the answer choice and finding that it weakens the argument, which sounds to me like the Assumption Negation Technique. This technique is fantastic, but it should only be applied to Assumption questions and, like you mentioned, this is a Justify question!

Instead, take a mechanistic approach for the questions. If you take the mechanistic approach to answer choice (D) instead, it will be much easier to see why (D) is incorrect. The new element in the conclusion but not the premises is the idea of "restriction", so we know that our correct answer choice must include some reference to the idea of restriction. This is the case because the conclusion must follow from the premises, so any new elements in the conclusion must be referenced in the answer choice to allow the conclusion to follow. (D) does not reference restrictions on content or connect it to censorship, so we can actually eliminate (D) without any further consideration.

Using the same approach, we see that (E) links the "restriction" to censorship in a way that allows the conclusion from the stimulus to follow, so we can select this answer choice!

I hope this helps :)
Kate

Get the most out of your LSAT Prep Plus subscription.

Analyze and track your performance with our Testing and Analytics Package.